Đến nội dung

gogo123

gogo123

Đăng ký: 25-09-2012
Offline Đăng nhập: 06-01-2024 - 22:50
*****

#645508 Chuyên đề thú vị về giới hạn dãy số

Gửi bởi gogo123 trong 19-07-2016 - 11:53

Tự nhiên lục lại thấy cái chuyên đề viết dở năm cấp 3, tặng anh em diễn đàn mình tham khảo (chỉ sợ nó lỗi thời rồi) , nếu bạn nào quan tâm thì liên hệ vs mình qua facebook mình sẽ gửi bản word và bạn tìm hiểu + giúp mình hoàn thành nốt phần còn lại nhé.  :D

File gửi kèm




#439428 Tìm tất cả các đa thức hệ số thực $P(x)$ thỏa mãn: $$P(x)...

Gửi bởi gogo123 trong 31-07-2013 - 00:42

Chứng minh của Sally trên sai ở ngay bước chứng minh hệ số $a_n$ dẫn đến kết luân nghiêm sai.

Dạng này có thể sử dụng số phức.

Dễ thấy nếu đa thức $x_0 \in R$ thì $P(x)$ cũng có nghiệm $x_0^2$ và $(x_0-2)^2$. Do vây nếu $\left | x_0 \right |\neq 1$ thì đa thức $P(x)$ có vô hạn nghiệm.

(Do nếu ngược lai thì $P(x)$ có các nghiệm $1<x_0<x_0^2<x_0^4<...$ hoặc $1>x_0>x_0^2>x_0^4>...>0$ đôi một phân biệt ,vô lí)

Suy ra $\left | x_0 \right |=1$ và $x_0=1$ (nếu $x_0=-1$ thì $P(x)$ có nghiệm $(x_0-2)^2=9>1$ vô lí) hay $P(x)=(x-1)^k.Q(x)$ với $Q(x)$ là đa thức hệ số thực không có nghiệm thực.

Dễ thấy thay vào phương trình $P(x)$ suy ra $Q(x)Q(x+2)=Q(x^2)$.  (1)

Giả sử $Q(x)$ có các nghiệm phức $\alpha _k=a_k+i.b_k$.Dễ thấy $\left |\alpha _k  \right |=1$ vì nếu ngược lại thì $Q(x)$ cũng có vô hạn nghiệm phức, vô lí.

Do đó $a_k^2+b_k^2=1$ (suy ra $a_k,b_k <1$, mặt khác theo (1) ta có Q(x) cũng có nghiệm $(\alpha _k -2)^2$.

Mà $\left |(\alpha _k -2)^2  \right |=(a_k-2)^2+b_k^2>1$ do $a_k<1 \Rightarrow  (a_k-2)^2>1$, vô lí vì mọi nghiệm của $Q(x)$ đều có modun là 1.

Suy ra $Q(x)=c$ hay $P(x)=c.(x-1)^k$, thay vào phương trình dễ thấy $c=0$ hoặc $c=1$.

Kết luận $P(x)=0$ hoặc $P(x)=(x-1)^k$ (k là số tự nhiên bất kì)




#439114 Phát phần thưởng năm 2013

Gửi bởi gogo123 trong 29-07-2013 - 17:12

Em nhận được rồi thầy ạ

Hình gửi kèm

  • Hình0123.JPG



#438092 IMO 2013

Gửi bởi gogo123 trong 25-07-2013 - 15:19

Bài 4 có cách phương tích 

Gọi $L,L'$ là giao điểm của $AW$ với các đường trong (1) và (2) .

Phương tích suy ra $AL.AW=AN.AB$,$AL'.AW=AM.AC$ mà $AN.AB=AM.AC$ nên $L=L'$ hay A,L,W thẳng hàng .

Do $XY$ sông song với $O_1O_2$ (đường trung bình) nên $XY$ vuông góc với $AW$.

Gọi $H'$ là giao điểm cuả đường cao $AP$ với $XY$ suy ra 2 tam giác $AH'L$ và $AWP$ đông dạng.

Suy ra: $AH'.AP=AL.AW=AN.AB$ mà $AN.AB=AH.AP$ hay $H'=H$. Suy ra $X,Y,H$ thẳng hàng.

 

 

Hình gửi kèm

  • untitled.JPG



#437239 Phát phần thưởng năm 2013

Gửi bởi gogo123 trong 22-07-2013 - 20:25

Địa chỉ nhận GCN của em là : Lê Kim Bình, xóm 2 Bắc Lâm, Diễn Lâm, Diễn Châu, Nghệ An.




#435330 Cho tam giác đều $ABC$, $D$ là 1 điểm nằm trên cạnh...

Gửi bởi gogo123 trong 15-07-2013 - 01:42

Gọi $Q,P$ lần lượt là tâm đường tròn ngoại tiếp $ABI,ACJ$.

Trước hết ta chứng minh $(Q,A,J);(P,A,I)$ là hai bộ điểm thẳng hàng.

   Gọi AT là phân giác $\angle CAD$, suy ra $AT\perp AJ$ (phân giác 2 góc bù nhau)

   Ta cần chứng minh $AT \perp AQ$ hay $AT$ là tiếp tuyến của $(Q)$, thật vậy ta có:

      $\angle AIB =180^{\circ}- \angle IAB -\angle IBA = 180^{\circ}-(90^{\circ}-\frac{ \angle SAD}{2} )-60^{\circ} = \frac{ \angle SAD}{2} + 30 ^{\circ}$

    $\angle BAT =60^{\circ}-\frac{ \angle CAD}{2}=\frac{ \angle SAD}{2} + 30 ^{\circ}$

  Do đó: $\angle AIB= \angle BAT $ hay $AT$ là tiếp tuyến của $(Q)$ suy ra $A,Q,J$ thẳng hàng.

  Tương tự ta cũng có $I,A,P$ thẳng hàng.

Trở lại bài toán ta sẽ chứng minh $JA$ là phân giác $\angle IJE$, thật vậy:

Ta có: $\angle ABI =\angle ACJ=60^{\circ} $ suy ra hai dây cung bị chắn $AI : AJ = R_{(Q)}:R_{(P)}$ hay  $\frac{AI}{AJ}=\frac{AQ}{AP}$

   Suy ra tứ giác $JIQP$ nội tiếp $\Rightarrow \angle IJA= \angle APQ$

   Xét đường tròn $(P)$ có $PQ$ vuông góc với dây $AE$ nên $\angle APQ =\angle AJE = 0,5.Sđ(\widehat{AE})$

  Suy ra : $\angle IJA= \angle APQ = \angle AJE$

Suy ra  $JA$ là phân giác $\angle IJE$

Tương tự $IA$ là phân giác $\angle JIE$

Vậy $A$ là tâm đường tròn nội tiếp tam giác $JIE$ (đpcm)

 

Hình gửi kèm

  • TST Turkey.JPG



#435329 Balkan MO 2013

Gửi bởi gogo123 trong 15-07-2013 - 00:28

Bài 1.Gọi $T$ là tâm đường tròn nội tiếp tam giác $ABC$.

Ta sẽ chứng minh $\angle QLK = \angle KMP  \Rightarrow \angle CLK = \angle TBA$ (phụ nhau)

Hay ta sẽ chứng minh 2 tam giác $TAB$ và $CKL$ đồng dạng.

Thật vậy,

   $\frac {CL}{AD}= \frac{QC}{QA} ; \frac{CK}{BE} = \frac{NC}{NB}$ 

 $\Rightarrow \frac {CL}{CK} = \frac {AD.QC.NB} {BE.NC.QA}$

Ta lại có :

  $BI = \frac {BM^2}{BE};\frac {BT}{BI}= \frac {BR}{BM} \Rightarrow BT= \frac {BM.BR}{BE} =\frac {BN.QC}{BE}$

 Tương tự ta cũng có $AT= \frac {AP.AR}{AD} = \frac {AQ.CN}{AD} $

 Suy ra: $ \frac {BT}{AT} = \frac {BN.QC.AD} {BE.AQ.CN} = \frac{CL}{CK}$

Mặt khác hai cặp cạnh $\left ( TB,CK \right );\left ( TA;CL \right )$ song song nên $\angle LCK =\angle BTA $

Do đo 2 tam giác $CLK$ và $TBA$ đồng dạng. Ta có đpcm.

Hình gửi kèm

  • Balkan.JPG



#434932 Topic về tổ hợp, các bài toán về tổ hợp

Gửi bởi gogo123 trong 13-07-2013 - 01:59

Bài 18.Sử dụng nội suyLagrange và đa thức Chebysev.

$T_{n-1}(x)=\sum_{k=0}^{n-1} T_{n-1}(x_k)\prod_{i\neq k}\frac{x-x_i}{x_k-x_i}$

Xét hệ số cao nhất ta có :

 $2^{n-2}=\sum_{k=0}^{n-1} \frac{T_{n-1}(x_k)}{\prod_{i\neq k}(x_k-x_i)}$

Suy ra 

$2^{n-2}\leq \left |\sum_{k=0}^{n-1} \frac{T_{n-1}(x_k)}{\prod_{i\neq k}(x_k-x_i)}  \right | \leq \sum_{k=0}^{n-1} \frac{1}{\left |\prod_{i\neq k}(x_k-x_i)  \right |}=S_n$




#434930 Iran NMO 2013

Gửi bởi gogo123 trong 13-07-2013 - 01:13

Ngày 2.

Bài 1.(hình vẽ)

   Dễ thấy $\angle PBT=\angle TAB;\angle ATB=\angle KAP'$.

   Do đó để chứng minh kết quả bài toán cần chứng minh $\Delta ATB\sim \Delta KAP'(c.g.c)$.

Thật vậy do $\angle ATB=\angle KAP'$ nên chỉ cần chứng minh

$$\frac{AK}{AT} = \frac{AP'}{TB} = \frac{BP}{BT}$$

Mà tứ giác $KTPB$ nội tiếp nên $ \angle TPB = \angle AKT $ và $\angle TAK= \angle TBP$ nên hai tam giác $TPB$ và $TAK$.

Suy ra : $\frac{AK}{AT} = \frac{BP}{BT}$ (đpcm)

Bài 2. Chú ý ta có thể đồng thời tăng giá trị ở mỗi hàng (cột ,đchéo) lên một số nguyên k bất kì (k có thể âm).

Xét TH bảng $3*3$ ta có: Hàng 1,2,3 cộng thêm $H_1;H_2;H_3$, cột 1,2,3 cộng thêm $C_1;C_2;C_3$, đường chéo có hiệu -2,-1,0,1,2 cộng thêm $D_1,D_2,D_3,D_4,D_5$
Khi đó cần có  $M(i,j)=H_i+C_j+D_{i-j}$. Cho $M_i$ tăng thêm 1 đơn vị ta chỉ việc bién đổi phần $H,C,D$ cho phù hợp là thoã mãn bài toán,

Bài 3.  $a_{n+2}=\left [ \frac{2a_n}{a_{n+1}} \right ]+\left [ \frac{2a_{n+1}}{a_n} \right ] $

Dễ thấy trong hai số nghịch đảo có 1 sô không nhỏ hơn 1 nên $a_n \geq 2$ với mọi n.

Và nếu có $a_{n}=a_{n+1}$ thì $a_{n+2}=4$

 

 Nếu có 1 số $a_n=2$ thì $a_{n+2}=a_{n+1} + \left [ \frac{4}{a_{n+1}} \right ]$.

   Khi đó 

              TH1. $a_{n+1}=3 \Rightarrow a_{n+2}=4;a_{n+3}=3$,t/m

              TH2.$a_{n+1}=4 \Rightarrow a_{n+3}=3;a_{n+4}=4;a_{n+5}=3$,t/m

              TH3. $a_{n+1}>5$ thì $a_{n+2}=a_{n+1}$ $ \Rightarrow$  $a_{n+3}=4$,tiếp tục xử lí như TH $a_n \geq 3$

 

 Nếu  $a_n \geq 3$ với mọi n thì

      TH1. Tồn tại $a_n<a_{n+1}$ suy ra $\left [ \frac{2a_{n+1}}{a_n} \right ] \leq a_{n+1}-1$ và $\left [ \frac{2a_n}{a_{n+1}} \right ] <1 $. 

        Do đó $a_{n+2} \leq a_{n+1}$ nên dãy có số hạng $a_{n+1}$ là lớn nhất.

   Hay ta có nhân xét là nếu tồn tại $a_n<a_{n+1}$ thì mộ số hạng phía sau $a_{n+1}$ đều không lớn hơn  $a_{n+1}$ và dương nên tồn tại một dãy vô hạn số hạng liên tiếp bằng nhau ( cùng = 4), t/m

       TH2. $a_{n} \geq a_{n+1}$ với mọi $n$ thì tồn tại một dãy có vô số số hạng liên tiếp cùng bằng nhau và bằng 4,t/m.

Như vậy ta có đpcm

 

 

Hình gửi kèm

  • Iran1.JPG



#414186 Dãy số-Giới hạn Tuyển tập sưu tầm các bài toán từ Mathlinks.ro

Gửi bởi gogo123 trong 21-04-2013 - 21:55

Bài 16. Xét trên cả Radian và Degree ta đều có: $1 \geq cos(\alpha) \geq 0$ với mọi $0 \geq \alpha \geq 1$.

Suy ra: $0 \geq a_n \geq 1$.

Xét hàm số $f(x)=cosx$ trên $(0,1)$ ta có $f'(x)<0$ suy ra $a_{2k}$ và $a_{2k+1}$ là 2 dãy đơn điệu, mà dãy bị chạn nên tồn tại giới hạn của 2 dãy, giả sử là $A$ và $B$.Suy ra:

$A=cos(B)$

$B=cos(A)$

Suy ra: $A=cos(cosA)$, đến đây xét hàm $g(x)=x-cos(cosx)$ trên $(0,1)$ có đạo hàm lớn hơn 0 nên có nghiệm và duy nhất. Do đó $A=B$.Hay dãy đã cho có giới hạn hữu hạn

Bài 17. Em nghĩ là nếu có lời giải số học thì cũng chỉ là biến đổi và quy nạp thôi, và hiển nhiên thì nó tương đương với việc xét pt đặc trưng.

Bài 12''. Em đang nghĩ đến việc chuyển nó về một bài toán tổ hợp đếm rồi suy ra kết quả bằng cách tính khác.

Kiểu như là : Cho tập $k$ người thì $a_k$ sẽ bằng số cách chọn ra một nhóm người nào đó thõa mãn điều kiện T

                     Khi đó với tập $m$ người thì ta lại tiếp tục chọn thõa mãn tính chất T, khi đó ta sẽ tách tập $m$ người này thành các tập nhỏ và được kết quả $a_m$ chính bẳng tổng các tích nhỏ (số cách chọn ra k người từ m người)*(số cách chọn cho tập k người thõa mãn T) = $\sum (C_m^k.a_k)$

Bây giờ chỉ còn việc chọn T sao cho $a_0=2$ và tồn tại phép  tổng ở trên. 




#412685 Phủ đa giác bởi hình tròn

Gửi bởi gogo123 trong 14-04-2013 - 22:57

Trong hình lồi F thì nó $=max\left \{ d(A,B) : A,B\in F\right \}$.

Đặc biệt trong đa giác thì đường kính là khoảng cách xa nhất giẵ hai đỉnh




#412254 Hỏi 3 đường tròn bán kính 0.5 có phủ hết hình vuông có cạnh bằng 1 không?

Gửi bởi gogo123 trong 13-04-2013 - 18:08

Spoiler

Câu trả lời là không.

Ta sẽ chứng minh bài toán thông qua việc chứng minh chúng không phủ hết cạnh của hình vuông.(mấu chốt).

    Trước hết xét một $(O,R)$ và một góc vuông $Axy$ sao cho $(O,R)$ cắt hai cạnh của góc lần lượt tại $M;N$.

    Ta có: (Hình vẽ) $AM^2+AN^2=MN^2 \leq (2R)^2=4R^2$

    Mặt khác theo AM-GM thì $AM^2+AN^2 \geq \frac{(AM+AN)^2}{2}$

 $\Rightarrow AM+AN \leq 2\sqrt{2}R$.

Trở lại bài toán ban đầu, ta dựng các đường tròn bán kính $0,5$ có tâm là các đỉnh của hình vuông, suy ra 2 đường tròn bất kì trong số chúng có chung nhau nhiều nhất 1 đỉnh.

Mặt khác 3 đường tròn đã cho phủ hết 4 đỉnh nên mỗi đường tròn ta mới vẽ chứa ít nhất một tâm của các hình tròn đã cho.

Theo nguyên lí Dirichle (3 đường tròn ,4 điểm) thì tồn tại một tam là tiếp điểm của 2 đường tròn (trung điểm một cạnh).

Như vậy còn 3 cạnh hình vuông hoàn toàn chưa bị phủ (tổng độ dài của chúng là 3).

Mà chỉ còn 2 đường tròn nên theo nhận xét ban đâu thì chúng phủ nhiều nhất tổng độ dài $4\sqrt{2}R=2\sqrt{2}<3$  (Vô lí).

Vậy ta có đpcm.

 

Hình gửi kèm

  • Picture0929.jpg



#412109 Phủ đa giác bởi hình tròn

Gửi bởi gogo123 trong 12-04-2013 - 21:02

Chứng minh trực tiếp: Ta chỉ cần chứng minh giao của tât cả đường tròn tâm đỉnh đa giác bán kính $\frac{d}{\sqrt{3}}$ khác rỗng.

 ta thấy Chỉ cần chứng minh cho tam giác , phần còn lại suy ra từ định lí Helly.

Xét tam giác ABC có độ dài 3 cạnh không lớn hơn $d$ (Khi đó đường kính của tam giác này không lớn hơn $d$ theo t/c đường kính của tam giác)

Nếu nhọn: Chọn tâm đường tròn nội tiếp. ( Chú ý gs AB lớn nhât thì $R=\frac{AB}{2sinC} \leq  \frac{d}{\sqrt{3}}$)

Nếu tù: Chọn tâm là trung điểm cạnh dài nhất . ( Chú ý $a^2+b^2<c^2$ nên $m_C = \sqrt{\frac{a^2+b^2}{2}-\frac{c^2}{4}}  \leq \frac{d}{\sqrt{3}}$)




#409888 Chứng minh $3|n$.

Gửi bởi gogo123 trong 02-04-2013 - 12:38


Cho $n$-giác . Một số đường chéo của $n$-giác thỏa mãn 3 tính chất sau:
1) Không có 2 đường chéo nào cắt nhau (trong đoạn)
2) $n$-giác bị chia thành các tam giác
3) Số đường chéo xuất phát từ mỗi đỉnh đều là số chẵn ( có thể là 0 )
CMR: $3|n$.

 

Chứng minh:

Xét đa giác $A_1A_2...A_n$

Gọi deg$(A_i)$ là bậc của đỉnh $A_i$, dễ thấy deg$(A_i)=2+$ số đường chéo xuất phát từ $A_i$. (Số 2 ở đây là do tính cả 2 cạnh).

Ta có $n$-giác bị chia thành các tam giác cạnh là đường chéo không căt nhau nên ta có:

Số tam giác tạo thành $\times 180^o$ $=$ Tổng các góc trong n-giác $=(n-2).180^o$.

Suy ra : Số tam giác tạo thành là $n-2$.

Ta có: 

  - Mỗi cạnh của $n$-giác thuộc duy nhất 1 tam giác

  - Mỗi đường chéo được vẽ thuộc đúng 2 tam giác.

Suy ra : Tổng tất cả các cạnh của các tam giác $=n+2.$(số đường chéo)

Mặt khác $\sum deg(A_i)=2n+2.$(số đường chéo).

Suy ra: Tổng cạnh tam giác $=3.(n-2)=n+2.$(số đường chéo)$=\sum deg(A_i)-n$.

$\Rightarrow \sum deg(A_i)=4n-6$.

Ta lại có $deg(A_i)$ là số chẵn dương nên giả sử có $k$ đỉnh không có đường chéo nào thì $4n-6=\sum deg(A_i) \geq 2k+4.(n-k)=4n-2k \Rightarrow k \geq 3$

Vậy có ít nhất 3 đỉnh không có đường chéo nào xuất phát từ đỉnh đó.Giả sử đó đỉnh $A_i$ thì ta có tam giác tạo thành chứa đỉnh đó duy nhất là $A_{i-1}A_iA_{i+1}$.Như vậy ta có quyền loại các đỉnh này ra khỏi đa giác đã cho mà không làm mất đi tính vẽ được của đa giác.

Xóa đi 3 đỉnh $deg=2$ ta thu được $(n-3)$ giác vẫn thõa mãn tính chất bài toán.Tiếp tục thực hiện như thế nhiều lần cho đến khi còn $\leq 3$  đỉnh ta sẽ thấy chỉ có khi còn lại 3 đỉnh thì n-giác ban đầu mới vẽ được như bài toán.

Vậy n chia hết cho 3.

Hình gửi kèm

  • VMF2.jpg



#409883 Chứng minh tồn tại 250 điểm mà 2 điểm bất kì cách nhau 1 khoảng $\g...

Gửi bởi gogo123 trong 02-04-2013 - 12:12

Dựng các đường tròn $(A_i;0,5)$ với $i=\overline{1,2011}$ ($A_i$ là các điểm).

Theo giả thiết bài toán các đường tròn này đôi một không cắt nhau.

Xét một điểm $A_i$ bất kì, lấy 6 điểm gần $A_i$ nhất.

Xem hình vẽ ta thấy có nhiều nhất sau đường tròn có thể cùng tiếp xúc với $(A_i;0,5)$. Khi đó đường tròn thứ 7 gần $(A_i;0,5)$ nhất là đường tròn $(B;0,5)$ như hình vẽ.Dễ dàng tính được $A_iB=\sqrt{3}$.

Như vậy tất cả các điểm khác 6 điểm gần $A_i$ nhất đều cách $A_i$ một khoảng không bé hơn $\sqrt{3}$.

Ta thực hiện như sau: Lấy một điểm bất kì rồi xóa đi 6 điểm gần nó nhất, xét các điểm không bị xóa khác điểm vừa chọn ta lại xóa đi 6 điểm gần nó nhất (6 điểm này không chứa điểm đã chọn ở bước trước),..., thực hiện thuật toán cho đến khi kết thúc ta thu được $\left [ \frac{2011}{6} \right ]$ điểm đôi một cách nhau $\geq \sqrt{3}$. 

Hình gửi kèm

  • VMF.jpg